SlideShare uma empresa Scribd logo
1 de 33
1
STANDARD COSTING-DETAILED ANALYSIS OF
MATERIAL VARIANVE
Introduction to Standard Costing
Standard costing is an important subtopic of cost accounting. Standard costs are usually
associated with a manufacturing company's costs of direct material, direct labor, and
manufacturing overhead.
Rather than assigning the actual costs of direct material, direct labor, and manufacturing
overhead to a product, many manufacturers assign the expected or standard cost. This means that
a manufacturer's inventories and cost of goods sold will begin with amounts reflecting the
standard costs, not the actual costs, of a product. Manufacturers, of course, still have to pay the
actual costs. As a result there are almost always differences between the actual costs and the
standard costs, and those differences are known as variances.
Standard costing and the related variances is a valuable management tool. If a variance arises,
management becomes aware that manufacturing costs have differed from the standard (planned,
expected) costs.
If actual costs are greater than standard costs the variance is unfavorable. An unfavorable
variance tells management that if everything else stays constant the company's actual profit will
be less than planned.
If actual costs are less than standard costs the variance is favorable. A favorable variance tells
management that if everything else stays constant the actual profit will likely exceed the planned
profit.
The sooner that the accounting system reports a variance, the sooner that management can direct
its attention to the difference from the planned amounts.
If we assume that a company uses the perpetual inventory system and that it carries all of its
inventory accounts at standard cost (including Direct Materials Inventory or Stores), then the
standard cost of a finished product is the sum of the standard costs of the inputs:
2
Direct material
Direct labor
Manufacturing overhead
Variable manufacturing overhead
Fixed manufacturing overhead
Usually there will be two variances computed for each input:
Since the calculation of variances can be difficult, we developed several business forms to help
you get started and to understand what the variances tell us.
Let's assume that your Uncle Pete runs a retail outlet that sells denim aprons in two sizes. Pete
suggests that you get into the manufacturing side of the business, so on January 1, 2013 you start
up an apron production company called Denim Works. Using the best information at hand, the
two of you compile the following estimates to use as standards for 2013:
The aprons are easy to produce, and no apron is ever left unfinished at the end of any given day.
This means that your company never has work-in-process inventory.
When we make your journal entries for completed aprons (shown below), we'll use an account
called Inventory-FG which means Finished Goods Inventory. (Some companies will use WIP
Inventory or Work-in-Process Inventory). We'll also use the account Direct Materials Inventory.
(Other account titles often used for direct materials are Raw Materials Inventory or Stores.)
Direct Materials Purchased: Standard Cost and Price Variance
Direct materials refer to just that—raw materials that are directly traceable into a product. In
your apron business the direct material is the denim. (In a food manufacturer's business the direct
materials are the ingredients such as flour and sugar; in an automobile assembly plant, the direct
materials are the cars' component parts).
Denim Works purchases its denim from a local supplier with terms of net 30 days, FOB
destination. This means that title to the denim passes from the supplier to Denim Works when
3
Denim Works receives the material. When the denim arrives, Denim Works will record the
denim received in its Direct Materials Inventory at the standard cost of rs3 per yard (see
standards table above) and will record the liability at the actual cost for the amount received.
Any difference between the standard cost of the material and the actual cost of the material
received is recorded as a purchase price variance.
Examples of Standard Cost of Materials and Price Variance
Let's assume that on January 2, 2013 Denim Works ordered 1,000 yards of denim at rs2.90 per
yard. On January 8, 2013 Denim Works receives 1,000 yards of denim and an invoice for the
actual cost of rs2,900. On January 8, 2013 Denim Works becomes the owner of the material and
has a liability to its supplier. On January 8 Denim Works' Direct Materials Inventory is increased
by the standard cost of rs3,000 (1,000 yards of denim at the standard cost of rs3 per yard),
Accounts Payable is credited for rs2,900 (the actual amount owed to the supplier), and the
difference of rs100 is credited to Direct Materials Price Variance. In general journal format the
entry looks like this:
The rs100 credit to the price variance account communicates immediately (when the denim
arrives) that the company is experiencing actual costs that are more favorable than the planned,
standard cost.
In February, Denim Works orders 3,000 yards of denim at rs3.05 per yard. On March 1, 2013
Denim Works receives the 3,000 yards of denim and an invoice for rs9,150 due in 30 days. On
March 1, the Direct Materials Inventory account is increased by the standard cost of rs9,000
(3,000 yards at the standard cost of rs3 per yard), Accounts Payable is credited for rs9,150 (the
actual cost of the denim), and the difference of rs150 is debited to Direct Materials Price
Variance as an unfavorable price variance:
After the March 1 transaction is posted, the Direct Materials Price Variance account shows a
debit balance of rs50 (the rs100 credit on January 2 combined with the rs150 debit on March 1).
A debit balance in a variance account is always unfavorable—it shows that the total of actual
costs is higher than the total of the expected standard costs. In other words, your company's
profit will be rs50 less than planned unless you take some action.
4
On June 1 your company receives 3,000 yards of denim at an actual cost of rs2.92 per yard for a
total of rs8,760 due in 30 days. The entry is:
Direct Materials Inventory is debited for the standard cost of rs9,000 (3,000 yards at rs3 per
yard), Accounts Payable is credited for the actual amount owed, and the difference of rs240 is
credited to Direct Materials Price Variance. A credit to the variance account indicates that the
actual cost is less than the standard cost.
After this transaction is recorded, the Direct Materials Price Variance account shows an overall
credit balance of rs190. A credit balance in a variance account is always favorable. In other
words, your company's profit will be rs190 greater than planned due to the favorable cost of
direct materials.
Note that the entire price variance pertaining to all of the direct materials received was recorded
immediately. In other words, the price variance associated with the direct materials received was
not delayed until the materials were used.
We will discuss later how to handle the balances in the variance accounts under the heading
"What To Do With Variance Amounts".
Direct Materials Usage Variance
Under a standard costing system, production and inventories are reported at the standard cost—
including the standard quantity of direct materials that should have been used to make the
products. If the manufacturer actually uses more direct materials than the standard quantity of
materials for the products actually manufactured, the company will have an unfavorable direct
materials usage variance. If the quantity of direct materials actually used is less than the standard
quantity for the products produced, the company will have a favorable usage variance. The
amount of a favorable and unfavorable variance is recorded in a general ledger account Direct
Materials Usage Variance. (Alternative account titles include Direct Materials Quantity Variance
or Direct Materials Efficiency Variance.) Let's demonstrate this variance with the following
information.
5
January 2013
In order to calculate the direct materials usage or quantity variance, we start with the number of
acceptable units of products that have been manufactured—also known as the good output. At
Denim Works this is the number of good aprons physically produced. If Denim Works produces
100 large aprons and 60 small aprons during January, the production and the finished goods
inventory will begin with the cost of the direct materials that should have been used to make
those aprons. Any difference will be a variance.
Note:
We are not determining the quantity of aprons that Denim Works should have made. Rather, we
are determining whether the 100 large aprons and 60 small aprons that were actually
manufactured were produced efficiently. In the case of direct materials, we want to determine
whether or not the company used the proper amount of denim to make the 160 aprons that were
actually produced. (For the purposes of calculating the direct materials usage variance, it does
not concern us whether Denim Works had a goal to produce 100 aprons, 200, aprons, or 250
aprons.)
Standard costs are sometimes referred to as the "should be costs." Denim Works should be using
278 yards of denim to make 100 large aprons and 60 small aprons as shown in the following
table.
We determine the total standard cost of the denim that should have been used to make the 160
aprons by multiplying the standard quantity of denim (278 yards) by the standard cost of a yard
of denim (rs3 per yard):
6
An inventory account (such as F.G. Inventory or Work-in-Process) is debited for rs834; this is
the standard cost of the direct materials component in the aprons manufactured in January 2013.
The Direct Materials Inventory account is reduced by the standard cost of the denim actually
removed from the direct materials inventory. Let's assume that the actual quantity of denim
removed from the direct materials inventory and used to make the aprons in January was 290
yards. Because Direct Materials Inventory reports the standard cost of the actual materials on
hand, we reduce the account balance by rs870 (290 yards used rs3 standard cost per yard). After
removing 290 yards of materials, the balance in the Direct Materials Inventory account is rs2,130
(710 yards x rs3 standard cost per yard).
The Direct Materials Usage Variance is: [the standard quantity of material that should have been
used to make the good output minus the actual quantity of material used] X the standard cost per
yard.
In our example, Denim Works should have used 278 yards of material to make 100 large aprons
and 60 small aprons. Because the company actually used 290 yards of denim, we say that Denim
Works did not operate efficiently—an extra 12 yards of denim was used (278 vs. 290 = 12).
When we multiply the 12 yards by the standard cost of rs3 per yard, the result is an unfavorable
direct materials usage variance of rs36.
Let's put the above information into a format commonly used for computing variances:
7
Direct Materials Usage/Quantity/Efficiency Variance Analysis
The journal entry for the direct materials portion of the January production is:
February 2013
Let's assume that in February 2013 Denim Works produces 200 large aprons and 100 small
aprons and that 520 yards of denim are actually used. From this information we can compute the
following:
8
Let's put the above information into our format:
Direct Materials Usage (or Quantity) Variance Analysis
The journal entry for the direct materials portion of the February production is:
9
Standard Costing
Standard costing is the practice of substituting an expected cost for an actual cost in the
accounting records, and then periodically recording variances that are the difference between the
expected and actual costs. This approach represents a simplified alternative to cost layering
systems, such as the FIFO and LIFO methods, where large amounts of historical cost information
must be maintained for items held in stock.
Standard costing involves the creation of estimated (i.e., standard) costs for some or all activities
within a company. The core reason for using standard costs is that there are a number of
applications where it is too time-consuming to collect actual costs, so standard costs are used as a
close approximation to actual costs.
Since standard costs are usually slightly different from actual costs, the cost accountant
periodically calculates variances that break out differences caused by such factors as labor rate
changes and the cost of materials. The cost accountant may also periodically change the standard
costs to bring them into closer alignment with actual costs.
Advantages of Standard Costing
Though most companies do not use standard costing in its original application of calculating the
cost of ending inventory, it is still useful for a number of other applications. In most cases, users
are probably not even aware that they are using standard costing, only that they are using an
approximation of actual costs. Here are some potential uses:
Budgeting: A budget is always composed of standard costs, since it would be impossible to
include in it the exact actual cost of an item on the day the budget is finalized. Also, since a key
application of the budget is to compare it to actual results in subsequent periods, the standards
used within it continue to appear in financial reports through the budget period.
Inventory costing: It is extremely easy to print a report showing the period-end inventory
balances (if you are using a perpetual inventory system), multiply it by the standard cost of each
item, and instantly generate an ending inventory valuation. The result does not exactly match the
actual cost of inventory, but it is close. However, it may be necessary to update standard costs
frequently, if actual costs are continually changing. It is easiest to update costs for the highest-
10
dollar components of inventory on a frequent basis, and leave lower-value items for occasional
cost reviews.
Overhead application: If it takes too long to aggregate actual costs into cost pools for allocation
to inventory, then you may use a standard overhead application rate instead, and adjust this rate
every few months to keep it close to actual costs.
Price formulation: If a company deals with custom products, then it uses standard costs to
compile the projected cost of a customer’s requirements, after which it adds on a margin. This
may be quite a complex system, where the sales department uses a database of component costs
that change depending upon the unit quantity that the customer wants to order. This system may
also account for changes in the company’s production costs at different volume levels, since this
may call for the use of longer production runs that are less expensive.
Nearly all companies have budgets and many use standard cost calculations to derive product
prices, so it is apparent that standard costing will find some uses for the foreseeable future. In
particular, standard costing provides a benchmark against which management can compare
actual performance.
Problems with Standard Costing
Despite the advantages just noted for some applications of standard costing, there are
substantially more situations where it is not a viable costing system. Here are some problem
areas:
Cost-plus contracts: If you have a contract with a customer under which the customer pays you
for your costs incurred, plus a profit (known as a cost-plus contract), then you must use actual
costs, as per the terms of the contract. Standard costing is not allowed.
Drives inappropriate activities: A number of the variances reported under a standard costing
system will drive management to take incorrect actions to create favorable variances. For
example, they may buy raw materials in larger quantities in order to improve the purchase price
variance, even though this increases the investment in inventory. Similarly, management may
schedule longer production runs in order to improve the labor efficiency variance, even though it
is better to produce in smaller quantities and accept less labor efficiency in exchange.
11
Fast-paced environment: A standard costing system assumes that costs do not change much in
the near term, so that you can rely on standards for a number of months or even a year, before
updating the costs. However, in an environment where product lives are short or continuous
improvement is driving down costs, a standard cost may become out-of-date within a month or
two.
Slow feedback: A complex system of variance calculations are an integral part of a standard
costing system, which the accounting staff completes at the end of each reporting period. If the
production department is focused on immediate feedback of problems for instant correction, the
reporting of these variances is much too late to be useful.
Unit-level information: The variance calculations that typically accompany a standard costing
report are accumulated in aggregate for a company’s entire production department, and so are
unable to provide information about discrepancies at a lower level, such as the individual work
cell, batch, or unit.
The preceding list shows that there are a multitude of situations where standard costing is not
useful, and may even result in incorrect management actions. Nonetheless, as long as you are
aware of these issues, it is usually possible to profitably adapt standard costing into some aspects
of a company’s operations.
Standard Cost Variances
A variance is the difference between the actual cost incurred and the standard cost against which
it is measured. A variance can also be used to measure the difference between actual and
expected sales. Thus, variance analysis can be used to review the performance of both revenue
and expenses.
There are two basic types of variances from a standard that can arise, which are the rate variance
and the volume variance. Here is more information about both types of variances:
Rate variance: A rate variance (which is also known as a price variance) is the difference
between the actual price paid for something and the expected price, multiplied by the actual
quantity purchased. The “rate” variance designation is most commonly applied to the labor rate
variance, which involves the actual cost of direct labor in comparison to the standard cost of
12
direct labor. The rate variance uses a different designation when applied to the purchase of
materials, and may be called the purchase price variance or the material price variance.
Volume variance: A volume variance is the difference between the actual quantity sold or
consumed and the budgeted amount, multiplied by the standard price or cost per unit. If the
variance relates to the sale of goods, it is called the sales volume variance. If it relates to the use
of direct materials, it is called the material yield variance. If the variance relates to the use of
direct labor, it is called the labor efficiency variance. Finally, if the variance relates to the
application of overhead, it is called the overhead efficiency variance.
Thus, variances are based on either changes in cost from the expected amount, or changes in the
quantity from the expected amount. The most common variances that a cost accountant elects to
report on are subdivided within the rate and volume variance categories for direct materials,
direct labor, and overhead. It is also possible to report these variances for revenue.
It is not always considered practical or even necessary to calculate and report on variances,
unless the resulting information can be used by management to improve the operations or lower
the costs of a business. When a variance is considered to have a practical application, the cost
accountant should research the reason for the variance in considerable detail and present the
results to the responsible manager, perhaps also with a suggested course of action.
Standard Cost Creation
At the most basic level, you can create a standard cost simply by calculating the average of the
most recent actual cost for the past few months. In many smaller companies, this is the extent of
the analysis used. However, there are some additional factors to consider, which can significantly
alter the standard cost that you elect to use. They are:
Equipment age: If a machine is nearing the end of its productive life, it may produce a higher
proportion of scrap than was previously the case.
Equipment setup speeds: If it takes a long time to setup equipment for a production run, the cost
of the setup, as spread over the units in the production run, is expensive. If a setup reduction plan
is contemplated, this can yield significantly lower overhead costs.
13
Labor efficiency changes: If there are production process changes, such as the installation of
new, automated equipment, then this impact the amount of labor required to manufacture a
product.
Labor rate changes: If you know that employees are about to receive pay raises, either through a
scheduled raise or as mandated by a labor union contract, then incorporate it into the new
standard. This may mean setting an effective date for the new standard that matches the date
when the cost increase is supposed to go into effect.
Learning curve: As the production staff creates an increasing volume of a product, it becomes
more efficient at doing so. Thus, the standard labor cost should decrease (though at a declining
rate) as production volumes increase.
Purchasing terms: The purchasing department may be able to significantly alter the price of a
purchased component by switching suppliers, altering contract terms, or by buying in different
quantities.
Any one of the additional factors noted here can have a major impact on a standard cost, which is
why it may be necessary in a larger production environment to spend a significant amount of
time formulating a standard cost.
14
FORMULA
Direct Materials Variances:
Materials purchase price variance Formula:
Materials purchase price variance = (Actual quantity purchased × Actual price) – (Actual
quantity purchased × Standard price)
Materials price usage variance formula
Materials price usage variance = (Actual quantity used × Actual price) – (Actual quantity used ×
Standard price)
materials quantity / usage variance formula
Materials price usage variance = (Actual quantity used × Standard price) – (Standard quantity
allowed × Standard price)
Materials mix variance formula
(Actual quantities at individual standard materials costs) – (Actual quantities at weighted average
of standard materials costs)
Materials yield variance formula
(Actual quantities at weighted average of standard materials costs) – (Actual output quantity at
standard materials cost)
15
MATERIAL COST VARIANCE
(SQ*SP) - (AQ*AP)
STD COST – ACT COST
MATERIAL USAGE VARIANCE
(SQ*SP) - (AQ*SP)
(SQ-AQ) * SP
MATERIAL PRICE VARIANCE
(SP*AQ) - (AP*AQ)
(SP-AP) *AQ
MATERIAL MIX VARIANCE
𝑇𝑄 𝑜𝑓 𝑎𝑐𝑡𝑢𝑎𝑙 𝑚𝑖𝑥
𝑇𝑄 𝑜𝑓 𝑠𝑡𝑎𝑛𝑑𝑎𝑟𝑑 𝑚𝑖𝑥
∗ 𝑠𝑡𝑎𝑛𝑑𝑎𝑟𝑑 𝑄 𝑜𝑓 𝑥
MATERIAL YEILD VARIANCE
(SP*SQ) – (SP*RQ)
(SQ-RQ) *SP
I.e. MCV = MUV + MPV
MCV = MMV + MYV
16
Materialcostvariance
Learning Objectives
To know how to calculate material cost variance
To know how to calculate material usage variance
To know how to calculate material price variance
Terminology for Material Variance Calculation
SQ =Standard Quantity
SR = Standard Rate
AQ = Actual Quantity
AP = Actual Price
RSQ = Resaved STD Quantity
MCV = Material Cost Variance
MUV = Material Usage Variance
MPV = Material Price Variance
MMV = Material Mix Variance
MYV = Material Yield Variance
17
Problem 1
The following information is obtained from X Co.
Ltd.Product SQ SP AQ AP(Units) (Rs.) (Units) (Rs.)A 1050 2.00 1100 2.25B 1500 3.25 1400 3.
50C 2100 3.50 2000 3.75Calculate the following:
Material cost variance
Material price variance
Material usage variance
Solution
1. MCV = (SQ x SR) – (AQ x AR)A = (1050 x 2.00) – (1100 x 2.25)= 2100 –
2475 = Rs. 375 (A)B = (1500 x 3.25) – (1400 x 3.50)= 4875 – 4900 = 25 (A)
2C = (2100 x 3.50) – (2000 x 7.25)= 7350 – 7500 = 150 (A)--------------------550 A----------------
----2. MPV = AQ (SP-AP)A = 1100(2.00 – 2.25) = 1100 x (0.25) = Rs. 275 (A)B = 1400(3.25 –
3.50) = 1400 x (-0.25) = Rs. 350 (A)C = 2000(3.50 – 3.75) = 2000 x (-0.25) = Rs. 500 (A)-------
----------1125 A-----------------3. MUV = SP (SQ-AQ)A = 2 (1050 -1100) = 2 (-
50) = Rs. 100 (A)B = 3.25 (1500 – 1400) = 3.25 (100) = Rs. 325 (F)C = 3.50 (2100 -
2000) = 3.50 (100) = Rs. 350 (F)---------------575 (F)---------------
Verification
MCV = MPV + MUVRs. 550 (A) = 1125(A) + 575 F550 (A) = 550 (A)
Problem 2
Product SQ SP Total AQ AP TotalA 10 2 20 5 3 15B 20 3 60 10 6 60C 20 6 120 15 5 75--- ----- -
-- -----Total 50 200 30 150--- ----- --- -----Find out (1) MCV (2) MPV (3) MMV (4) MUV
Sol.:
(1) MCV = (SQ x SR) – (AQ x AR)A = (10 x 2) – (5 x 3) = 5 FB = (20 x 3) – (10 x 6) = Nil
18
Direct Material Price Variance
Direct material price variance (also called the direct material spending/rate variance) is the
product of actual quantity of direct material used and the difference between standard price and
actual price per unit of direct material. It is calculated using the following formula:
DM Price Variance = (SP − AP) × AQ
Where,
SP is the standard unit price of direct material
AP is the actual price per unit of direct material
AQ is the actual quantity of direct material used
Analysis
Direct material price variance is calculated to determine the efficiency of purchasing department
in obtaining direct material at low cost. A positive value of direct material price variance is
favorable which means that direct material was purchased for lesser amount than the standard
price. A negative value of direct material price variance is unfavorable because more than
estimated price per unit is paid.
However, a favorable direct material price variance is not always good; it should be analyzed
together with direct material quantity variance. It is quite possible that the purchasing department
may purchase low quality raw material to generate a favorable direct material price variance.
Such a favorable material price variance will be offset by an unfavorable direct material quantity
variance due to wastage of low quality direct material.
Example
Calculate the direct material price variance if the standard price and actual unit price per unit of
direct material are RS4.00 and RS4.10 respectively; and actual units of direct material used
during the period are 1,200. Determine whether the variance is favorable or unfavorable.
19
Standard Price RS4.00
− Actual Price 4.10
Difference Per Unit − 0.10
× Actual Quantity 1,200
Direct Material Price Variance − RS 120
Since the price paid by the company for the purchase of direct material is more than the standard
price by RS120, the DM price variance is unfavorable.
In managerial accounting, variance means deviation of actual costs from standard costs.
Materials price variance is the result of deviation of actual price paid for materials from what has
been set as standard. Direct materials price and quantity standards are set after keeping in mind
the current market prices and anticipated changes in materials prices in near future. However
things do not always happen as expected. The actual price of materials may significantly deviate
from standard price. Moreover, the expenses associated with the order (freight, duties, handling
expenses etc.) may increase or reduce the price of materials available for use. The business may
have to pay more or less price than what has been considered as normal at the time of setting
standards.
If the actual price paid for materials is more than the standard price, an unfavorable materials
price variance occurs. On the other hand, if the actual price paid for the materials is less than the
standard price, a favorable materials price variance occurs.
An example is given below for more explanation:
Example
The Aptex Company manufactures and sells small speakers that are used in mobile phones. The
speakers are sold in bulk to mobile manufacturing companies where complete mobiles are
produced. The direct material of Aptex Company is a thin copper coil. One meter of the copper
coil is the standard requirement to manufacture one speaker.
The standard cost to manufacture one speaker is as follows:
20
Direct materials: 1meters × RS1.50 per meters RS1.50
Direct labor RS1.00
Manufacturing overhead RS0.50
—–
Total RS3.00
—–
During the month of June, Aptex purchased 5,000 meters of copper coil @ RS1.70 per meter and
produced 2,500 speakers using 3,000 meters of copper coil.
The materials price variance for the month of June can be calculated using the following
formula/equation:
= (5,000 × RS1.70) – (5,000 × RS1.50)
= RS8,500 – RS7,500
= 1,000 Unfavorable
Aptex has an unfavorable materials price variance for June because the actual price paid
(RS8,500) is more than the standard price allowed (RS7,500) for 5,000 meters of copper coil.
This variance can also be computed by using the factored form of above formula:
AQ × (AP – SP)
5,000 meters × (RS1.70 – RS1.50)
5,000 meters × RS0.20
RS1,000 Unfavorable
21
Reasons of direct materials price variance:
A favorable or unfavorable material price variance may occur due to one or more of the
following reasons:
Order size: Some suppliers allow discount on large orders. The materials purchased in large
quantities may reduce the unit price and a favorable price variance may occur.
Rise in price: The rise in the general price level may increase the input costs of the vendor and as
a result vendor may increase the price of the materials. The rise in price is very common reason
of an unfavorable variance.
Urgent needs: If production department does not indicate the need of materials on time, the
purchasing department may have to order on urgent basis that may increase the price of materials
and other expenses associated with the order.
Quality: A favorable price variance may be the result of purchasing low quality materials and an
unfavorable variance may be the result of purchasing high quality materials.
Inefficient standard setting: Inefficiencies in terms of forecasting and environmental scanning
during standard setting process can be a reason of huge variances.
Transportation: Transportation is a part of total direct materials cost. Any change in the
transportation expenses can change the total and per unit cost of direct materials available for use
and can become the reason of favorable or unfavorable direct materials price variance.
The role of just in time manufacturing: A company that operates under just in time (JIT)
manufacturing system may have to face shortage of direct materials due to a sudden increase in
demand for the product. The orders in rush normally increase the costs. In that case company
will have to either accept an unfavorable materials price variance or lost sales.
Inefficient or unreliable Suppliers: A deviation from standard material costs may be the result of
inefficient or unreliable vendors. For example, if suppliers of raw materials are unable to meet
the demand, the company may have to look for another supplier who may be more costly.
22
Responsibility of the variance:
Purchasing department is responsible to place orders for direct materials so this variance is
generally considered the responsibility of purchase manager. However, the above reasons clarify
that the materials price variance may or may not be the result of inefficiencies of the purchasing
department.
The occurrence of variances is very normal. They occur for almost all cost elements and should
not be used to find someone to blame. Sometimes they may not be very significant and
sometimes they may be the result of the factors that are beyond the control of managers.
Variances are tools to control costs, improve efficiencies and should be used positively.
Direct Material Quantity Variance
Direct material quantity variance (also called the direct material usage/efficiency variance) is the
product of standard price of a unit of direct material and the difference between standard quantity
of direct material allowed and actual quantity of direct material used. The formula to calculate
direct material quantity variance is:
DM Quantity Variance = ( SQ − AQ ) × SP
Where,
SQ is the standard quantity allowed
AQ is the actual quantity of direct material used
SP is the standard price per unit of direct material
Standard quantity allowed (SQ) is calculated as the product of standard quantity of direct
material per unit and actual units produced.
Analysis
Direct material quantity variance is calculated to determine the efficiency of production
department in converting raw material to finished goods. In order to improve efficiency, wastage
of raw material must be reduced. A negative value of direct material quantity variance is
23
unfavorable and it implies that more quantity of direct material has been used in the production
process than actually needed. A positive value of direct material quantity variance is favorable
implying that raw material was efficiently converted to finished goods.
Example
Use the following information to calculate direct material quantity variance. Also specify
whether the variance is favorable or unfavorable.
Standard Price of a Unit of Direct Material RS4
Standard Quantity of Direct Material Per Unit 2
Actual Units Produced During the Period 620
Actual Quantity Used During the Period 1,200
Solution
Actual Units Produced 620
× Standard Quantity of Direct Material Per Unit 2
Standard Quantity Allowed 1,240
Standard Quantity Allowed 1,240
− Actual Quantity 1,200
Difference 40
× Standard Price of a Unit of Direct Material RS4
Direct Material Quantity Variance RS160
In this case the production department performed efficiently and saved 40 units of direct
material. Multiplying this by standard price per unit yields a favorable direct material quantity
variance of RS160.
24
Direct Material Usage Variance
Direct Material Usage Variance is the measure of difference between the actual quantity of
material utilized during a period and the standard consumption of material for the level of output
achieved.
Direct Material Usage Variance:
= Actual Quantity x Standard Price - Standard Quantity x Standard Price
= Standard Cost of Actual Quantity - Standard Cost of Standard Quantity
= (Actual Quantity - Standard Quantity) x Standard Price
Since the effect of any variation in material price from the standard is calculated in the material
price variance, material usage variance is calculated using the standard price.
Example
Cement PLC manufactured 10,000 bags of cement during the month of January. Consumption of
raw materials during the period was as follows:
Material Quantity Used Standard Usage Per Bag Actual Price Standard Price
Limestone 100 tons 11 KG RS75/ton RS70/ton
Clay 150 tons 14 KG RS21/ton RS20/ton
Sand 250 tons 26 KG RS11/ton RS10/ton
Material Usage Variance will be calculated as follows:
Step 1: Calculate Standard Quantity
Limestone: 10,000 units x 11 / 1000 = 110 tons
Clay: 10,000 units x 14 / 1000 = 140 tons
Sand: 10,000 units x 26 / 1000 = 260 tons
25
Step 2: Calculate the Variance
Material Usage Variance = [Actual Quantity - Standard Cost (Step 2)] x Standard Price
Limestone: (100 - 110) x RS70 = (RS700) Favorable
Clay: (150 - 140) x RS20 = RS200 Adverse
Sand: (250 - 260) x RS10 = (RS100) Favorable
Total Usage Variance (RS600) Favorable
Note: Actual price paid for the acquisition of materials shall be ignored since the variation
between standard price is already accounted for in the material price variance.
Analysis
A favorable material usage variance suggests efficient utilization of materials.
Reasons for a favorable material usage variance may include:
Purchase of materials of higher quality than the standard (this will be reflected in adverse
material price variance)
Greater use of skilled labor
Training and development of workforce to improve productivity
Use and improvement of automated manufacturing tools and processes
An adverse material usage variance indicates higher consumption of material during the period
as compared with the standard usage.
Reasons for adverse material usage variance include:
Purchase of materials of lower quality than the standard
Use of unskilled labor
Increase in material wastage due to depreciation of plant and equipment
26
Direct Material Mix Variance
Definition
Direct Material Mix Variance is the measure of difference between the cost of standard
proportion of materials and the actual proportion of materials consumed in the production
process during a period.
Direct Material Mix Variance:
= Actual Quantity x Standard Price - Standard Mix Quantity x Standard Price
= Standard Cost of Actual Mix - Standard Cost of Standard Mix
= (Actual Mix Quantity - Standard Mix Quantity) x Standard Price
As material mix variance is an extension of the material usage variance, the variance is based on
the standard price rather than actual price since the difference between actual and standard
material price is accounted for separately in the material price variance.
Example
Cement PLC manufactured 10,000 bags of cement during the month of January. Consumption of
raw materials during the period was as follows:
Material Quantity UsedStandard Mix Per BagActual Price Standard Price
Limestone 100 tons 11 KG RS75/ton RS70/ton
Clay 150 tons 14 KG RS21/ton RS20/ton
Sand 250 tons 26 KG RS11/ton RS10/ton
27
Material Mix Variance will be calculated as follows:
Step 1: Calculate the total consumption of raw materials
Total Raw Materials Consumption (100 + 150 + 250) = 500 tons
Step 2: Calculate the Standard Mix
We need to calculate the quantity of each raw material which would have been consumed had the
total usage of raw materials (500 tons) been based on the standard mix.
Limestone: 500 tons units x11 / 51*=108 tons
Clay: 500 tons units x14 / 51*=137 tons
Sand: 500 tons units x26 / 51*=255 tons
* Total Quantity under Standard Usage (11 + 14 + 26) = 51 KG per bag
Note that the sum of the standard mix of raw materials calculated above equals the actual total
consumption of 500 tons. This is because in material mix variance, we are not concerned about
the efficiency of raw material consumption but rather their relevant proportions.
Step 3: Calculate the Variance
Material Usage Variance = [Actual Mix - Standard Mix (Step 2)] x Standard
Price
Limestone: (100 - 108) x RS70 = (RS560) Favorable
Clay: (150 - 137) x RS20 = RS260 Adverse
Sand: (250 - 255) x RS10 = (RS50) Favorable
Total Usage Variance (RS350) Favorable
28
Explanation
Material Mix Variance quantifies the effect of a variation in the proportion of raw materials used
in a production process over a period.
Material mix variance is a sub-division of material usage variance. While material usage
variance illustrates the overall efficiency of raw material consumption during a period (in terms
of the difference between the amount of materials which should have been used and the actual
usage), material mix variance focuses on the aspect of proportion of raw materials used in the
production process.
Material mix variance is only suitable for performance measurement and control where the
proportion of inputs to the production process can be altered without reducing the effectiveness
of the final product. It may not therefore be used in industries that require a high degree of
precision in the input variables such as in the pharmaceuticals sector.
Analysis
A favorable material mix variance suggests the use of a cheaper mix of raw materials than the
standard. Conversely, an adverse material mix variance suggests that a more costly combination
of materials have been used than the standard mix.
A change in the material mix must also be analyzed in the context of other organization wide
implications that may follow. Some of the effects a change in direct material mix include:
 Change in the quality, performance and durability of the final product
 Price offered by customers may vary as a result of a change in perceived quality of the
product
 Change in material mix may affect the workability of materials which may in turn affect
labor efficiency
29
Direct Material Yield Variance
Definition
Direct Material Yield Variance is a measure of cost differential between output that should have
been produced for the given level of input and the level of output actually achieved during a
period.
Direct Material Yield Variance:
= (Actual Yield - Standard Yield) x Standard Material Cost Per Unit
Example
Cement PLC manufactured 10,000 bags of cement during the month of January. Consumption of
raw materials during the period was as follows:
Material Quantity UsedStandard Mix Per BagActual Price Standard Price
Limestone 100 tons 11 KG RS75/KG RS70/KG
Clay 150 tons 14 KG RS21/KG RS20/KG
Sand 250 tons 26 KG RS11/KG RS10/KG
Material Yield Variance shall be calculated as follows:
Step 1: Calculate the Standard Yield for the total materials input
500 tons of materials should have yielded 9,804 bags
Standard Yield = 500 tons x 1000 / 51 KG = 9,804 bags
30
Step 2: Calculate the Standard Cost of materials per bag
Total material cost of 1 bag of cement:
Limestone: 11 KGxRS70 =RS770
Clay: 14 KGxRS20 =RS280
Sand: 26 KGxRS10 =RS260
Total RS1,310 per bag
Actual material price should be ignored since the variance between actual and standard price is
accounted for in the material price variance.
Step 3: Calculate the Total Yield Variance
Material Usage Variance = [Actual Yield - Standard Yield (Step 1)] x Standard Cost / Unit
(Step 2)
Actual Yield - Standard Yield = 10,000 - 9,804 (Step 1) = 196 bags
Total Material Yield Variance = 196 bags x RS1,310 (Step 2)
= RS256,760 Favorable
As the actual output achieved during the period is higher than the standard yield, the variance is
favorable. Favorable material yield variance indicates the amount of savings in material costs as
a result of better output yield than the standard.
31
Step 4: Calculate the Material Wise Yield Variances
Individual material yield variance can be calculated in a similar way to the total yield variance.
Materials:
Actual Yield - Standard Yield
(Step 3)
x
Standard Cost per bag
(Step 2)
=Yield Variance
Limestone: 196 bags xRS770 =RS150,920
Clay: 196 bags xRS280 =RS54,880
Sand: 196 bags xRS260 =RS50,960
RS256,760
Note that sum of individual material yield variances equals the total yield variance calculated in
step 3.
Explanation
Material Yield Variance measures the effect on material cost of a change in the production yield
from the standard.
Material yield variance is used in conjunction with material mix variance in order to provide
additional analysis of the material usage variance.
The difference between material usage and material yield variance is that the former focuses on
the utilization of input at the start of production process whereas latter focuses on the efficiency
in terms of the output yield during a period.
Analysis
A favorable material yield variance indicates better productivity than the standard yield resulting
in lower material cost.
Conversely, an adverse material yield variance suggests lower production achieved during a
period for the given level of input resulting in higher material cost.
32
Conclusion
Standard costing is a technique used for the purpose of determining standard cost and their
comparison with the actual cost to find out the cause of difference between the two so that
remedial action may be taken immediately.
Direct material cost variance is the difference between the standard cost of materials specified
for the output achieved, and the actual cost of direct materials consumed.
Direct material price variance is the difference between actual price and standard price of
materials consumed.
Material usage variance is that portion of material cost which arises due to the difference
between the standard quantity specified and the actual quantity used.
Material mix variance may be defined as that portion of the material usage variance which is due
to the difference between the standard and actual composition of material mixture.
Material yield variance is calculated on the basis of output while the other variance are
calculated on the basis of input. The variance is calculated as the difference between the standard
output and actual output.
33
Reference
http://www.accountingcoach.com/standard-costing/explanation/1
http://www.accountingcoach.com/standard-costing/explanation/2
http://www.accountingtools.com/standard-costing
http://www.accounting4management.com/variance_formulas.htm
http://accountingexplained.com/managerial/standard-costing/dm-price-variance
http://www.accountingformanagement.org/direct-materials-price-variance/
http://accounting-simplified.com/management/variance-analysis/material/usage.html
http://accountingexplained.com/managerial/standard-costing/dm-quantity-variance
http://accounting-simplified.com/management/variance-analysis/material/mix.html
http://accounting-simplified.com/management/variance-analysis/material/yield.html
https://www.scribd.com/doc/6988338/Material-Cost-Variance-Analysis

Mais conteúdo relacionado

Mais procurados

Relevant information and decision making
Relevant information and decision makingRelevant information and decision making
Relevant information and decision makingAliza Racelis
 
Standard costs and variance analysis
Standard costs and variance analysisStandard costs and variance analysis
Standard costs and variance analysisSumit Malhotra
 
A presentation on process costing
A presentation on process costingA presentation on process costing
A presentation on process costingPRIYANKAVP4
 
MBA 628 Management Accounting Case Study
MBA 628 Management Accounting Case StudyMBA 628 Management Accounting Case Study
MBA 628 Management Accounting Case Studymarhenbun
 
Absorption costing vs variable costing
Absorption costing  vs variable costingAbsorption costing  vs variable costing
Absorption costing vs variable costingShankar Jyoti Doley
 
Process costing
Process costingProcess costing
Process costingsigh_baba
 
Joint product and by product costing
Joint product and by product costingJoint product and by product costing
Joint product and by product costingMuhammad Usman
 
Standard costing
Standard costingStandard costing
Standard costingzaidul2
 
Chapter 5 : Relevant Costing For Decision Making
Chapter 5 : Relevant Costing For Decision MakingChapter 5 : Relevant Costing For Decision Making
Chapter 5 : Relevant Costing For Decision MakingPeleZain
 
Overhead cost variance
Overhead cost varianceOverhead cost variance
Overhead cost variancePAWANGUPTA327
 
Waiting Line Management
Waiting Line Management Waiting Line Management
Waiting Line Management Joshua Miranda
 

Mais procurados (20)

PROCESS-COSTING
PROCESS-COSTING PROCESS-COSTING
PROCESS-COSTING
 
Relevant information and decision making
Relevant information and decision makingRelevant information and decision making
Relevant information and decision making
 
Standard costs and variance analysis
Standard costs and variance analysisStandard costs and variance analysis
Standard costs and variance analysis
 
Job costing 3 a
Job costing 3 aJob costing 3 a
Job costing 3 a
 
A presentation on process costing
A presentation on process costingA presentation on process costing
A presentation on process costing
 
MBA 628 Management Accounting Case Study
MBA 628 Management Accounting Case StudyMBA 628 Management Accounting Case Study
MBA 628 Management Accounting Case Study
 
Absorption costing vs variable costing
Absorption costing  vs variable costingAbsorption costing  vs variable costing
Absorption costing vs variable costing
 
Process costing
Process costingProcess costing
Process costing
 
Joint product and by product costing
Joint product and by product costingJoint product and by product costing
Joint product and by product costing
 
Standard costing
Standard costingStandard costing
Standard costing
 
Chapter 5 : Relevant Costing For Decision Making
Chapter 5 : Relevant Costing For Decision MakingChapter 5 : Relevant Costing For Decision Making
Chapter 5 : Relevant Costing For Decision Making
 
Cost accounting
Cost accountingCost accounting
Cost accounting
 
Uniform costing
Uniform costingUniform costing
Uniform costing
 
Standard costing
Standard costingStandard costing
Standard costing
 
Overhead cost variance
Overhead cost varianceOverhead cost variance
Overhead cost variance
 
ch.31 process costing.ppt
ch.31 process costing.pptch.31 process costing.ppt
ch.31 process costing.ppt
 
Waiting Line Management
Waiting Line Management Waiting Line Management
Waiting Line Management
 
As 2
As 2As 2
As 2
 
Product costing
Product costingProduct costing
Product costing
 
Standard costing setting standards and analysis of variance
Standard costing setting standards and analysis of varianceStandard costing setting standards and analysis of variance
Standard costing setting standards and analysis of variance
 

Destaque

Stdcostingppt
StdcostingpptStdcostingppt
StdcostingpptTaha Rais
 
Budgetory Control System and Cost Control
Budgetory Control System and Cost ControlBudgetory Control System and Cost Control
Budgetory Control System and Cost ControlKishore Raveendran
 
Ca chap 13 standard costing&variance analysis(2)
Ca chap 13 standard costing&variance analysis(2)Ca chap 13 standard costing&variance analysis(2)
Ca chap 13 standard costing&variance analysis(2)DSDEVDA
 
Solution Manual Cost Accounting Planning and Control by Matz.Hammer and Usry ...
Solution Manual Cost Accounting Planning and Control by Matz.Hammer and Usry ...Solution Manual Cost Accounting Planning and Control by Matz.Hammer and Usry ...
Solution Manual Cost Accounting Planning and Control by Matz.Hammer and Usry ...Bushra Sultana Malik
 
Budgeting & Variance Analysis
Budgeting & Variance AnalysisBudgeting & Variance Analysis
Budgeting & Variance AnalysisSam walker
 
Cost Accounting
Cost AccountingCost Accounting
Cost Accountingashu1983
 

Destaque (7)

Stdcostingppt
StdcostingpptStdcostingppt
Stdcostingppt
 
Budgetory Control System and Cost Control
Budgetory Control System and Cost ControlBudgetory Control System and Cost Control
Budgetory Control System and Cost Control
 
Variance Analysis
Variance AnalysisVariance Analysis
Variance Analysis
 
Ca chap 13 standard costing&variance analysis(2)
Ca chap 13 standard costing&variance analysis(2)Ca chap 13 standard costing&variance analysis(2)
Ca chap 13 standard costing&variance analysis(2)
 
Solution Manual Cost Accounting Planning and Control by Matz.Hammer and Usry ...
Solution Manual Cost Accounting Planning and Control by Matz.Hammer and Usry ...Solution Manual Cost Accounting Planning and Control by Matz.Hammer and Usry ...
Solution Manual Cost Accounting Planning and Control by Matz.Hammer and Usry ...
 
Budgeting & Variance Analysis
Budgeting & Variance AnalysisBudgeting & Variance Analysis
Budgeting & Variance Analysis
 
Cost Accounting
Cost AccountingCost Accounting
Cost Accounting
 

Semelhante a STANDARD COSTING-DETAILED ANALYSIS OF MATERIAL VARIANCE

Cost of goods sold
Cost of goods soldCost of goods sold
Cost of goods soldAsad Khan
 
Jiambalvo text book solutions (1)
Jiambalvo text book solutions (1)Jiambalvo text book solutions (1)
Jiambalvo text book solutions (1)Mvs Krishna
 
How to calculate the cost of goods manufactured (COGM)?
How to calculate the cost of goods manufactured (COGM)?How to calculate the cost of goods manufactured (COGM)?
How to calculate the cost of goods manufactured (COGM)?MRPeasy
 
ACCOUNTING PRODMA BANA REVIEWER for class
ACCOUNTING PRODMA BANA REVIEWER for classACCOUNTING PRODMA BANA REVIEWER for class
ACCOUNTING PRODMA BANA REVIEWER for classJosephineTorres14
 
Material ledger by guntupalli hari krishna
Material ledger by guntupalli hari krishnaMaterial ledger by guntupalli hari krishna
Material ledger by guntupalli hari krishnaHari Krishna
 
WHY IS THE ALLOCATION METHOD USED IN ACCOUNTING FOR THE DIFFERENCE BETWEEN AP...
WHY IS THE ALLOCATION METHOD USED IN ACCOUNTING FOR THE DIFFERENCE BETWEEN AP...WHY IS THE ALLOCATION METHOD USED IN ACCOUNTING FOR THE DIFFERENCE BETWEEN AP...
WHY IS THE ALLOCATION METHOD USED IN ACCOUNTING FOR THE DIFFERENCE BETWEEN AP...Mashfiq Albartross
 
Standard price & moving average price in SAP
Standard price & moving average price in SAPStandard price & moving average price in SAP
Standard price & moving average price in SAPShalehur Rahman Samy
 
Cost of Goods Sold.docx
Cost of Goods Sold.docxCost of Goods Sold.docx
Cost of Goods Sold.docxoneformany
 
Cost Accounting Systems
Cost Accounting SystemsCost Accounting Systems
Cost Accounting SystemsItzAnomynous
 
Manufacturing Cost Reduction Project ppt
Manufacturing Cost Reduction Project pptManufacturing Cost Reduction Project ppt
Manufacturing Cost Reduction Project pptDeepak Joshi
 
How to Calculate Product Costs for a Manufacturer
How to Calculate Product Costs for a ManufacturerHow to Calculate Product Costs for a Manufacturer
How to Calculate Product Costs for a ManufacturerIsah Nurdianah
 
Accounting for Inventory
Accounting for InventoryAccounting for Inventory
Accounting for InventoryMang Engkus
 
Comparing Major Party PlatformsUsing the party platforms provide.docx
Comparing Major Party PlatformsUsing the party platforms provide.docxComparing Major Party PlatformsUsing the party platforms provide.docx
Comparing Major Party PlatformsUsing the party platforms provide.docxdonnajames55
 
ACCOUNTING FOR MATERIAL.pptx
ACCOUNTING FOR MATERIAL.pptxACCOUNTING FOR MATERIAL.pptx
ACCOUNTING FOR MATERIAL.pptxDaveN31
 

Semelhante a STANDARD COSTING-DETAILED ANALYSIS OF MATERIAL VARIANCE (20)

Cost of goods sold
Cost of goods soldCost of goods sold
Cost of goods sold
 
Jiambalvo text book solutions (1)
Jiambalvo text book solutions (1)Jiambalvo text book solutions (1)
Jiambalvo text book solutions (1)
 
How to calculate the cost of goods manufactured (COGM)?
How to calculate the cost of goods manufactured (COGM)?How to calculate the cost of goods manufactured (COGM)?
How to calculate the cost of goods manufactured (COGM)?
 
Costing system
Costing systemCosting system
Costing system
 
ACCOUNTING PRODMA BANA REVIEWER for class
ACCOUNTING PRODMA BANA REVIEWER for classACCOUNTING PRODMA BANA REVIEWER for class
ACCOUNTING PRODMA BANA REVIEWER for class
 
L and SCM Unit 3
L and SCM Unit 3L and SCM Unit 3
L and SCM Unit 3
 
Material ledger by guntupalli hari krishna
Material ledger by guntupalli hari krishnaMaterial ledger by guntupalli hari krishna
Material ledger by guntupalli hari krishna
 
WHY IS THE ALLOCATION METHOD USED IN ACCOUNTING FOR THE DIFFERENCE BETWEEN AP...
WHY IS THE ALLOCATION METHOD USED IN ACCOUNTING FOR THE DIFFERENCE BETWEEN AP...WHY IS THE ALLOCATION METHOD USED IN ACCOUNTING FOR THE DIFFERENCE BETWEEN AP...
WHY IS THE ALLOCATION METHOD USED IN ACCOUNTING FOR THE DIFFERENCE BETWEEN AP...
 
New perpetual inventory system
New perpetual inventory systemNew perpetual inventory system
New perpetual inventory system
 
Standard price & moving average price in SAP
Standard price & moving average price in SAPStandard price & moving average price in SAP
Standard price & moving average price in SAP
 
Cost of Goods Sold.docx
Cost of Goods Sold.docxCost of Goods Sold.docx
Cost of Goods Sold.docx
 
Cost Accounting Systems
Cost Accounting SystemsCost Accounting Systems
Cost Accounting Systems
 
Manufacturing Cost Reduction Project ppt
Manufacturing Cost Reduction Project pptManufacturing Cost Reduction Project ppt
Manufacturing Cost Reduction Project ppt
 
Sample powerpoint
Sample powerpointSample powerpoint
Sample powerpoint
 
How to Calculate Product Costs for a Manufacturer
How to Calculate Product Costs for a ManufacturerHow to Calculate Product Costs for a Manufacturer
How to Calculate Product Costs for a Manufacturer
 
Accounting for Inventory
Accounting for InventoryAccounting for Inventory
Accounting for Inventory
 
Inventory Modelling
Inventory ModellingInventory Modelling
Inventory Modelling
 
Comparing Major Party PlatformsUsing the party platforms provide.docx
Comparing Major Party PlatformsUsing the party platforms provide.docxComparing Major Party PlatformsUsing the party platforms provide.docx
Comparing Major Party PlatformsUsing the party platforms provide.docx
 
Brs3e online only-ch12
Brs3e online only-ch12Brs3e online only-ch12
Brs3e online only-ch12
 
ACCOUNTING FOR MATERIAL.pptx
ACCOUNTING FOR MATERIAL.pptxACCOUNTING FOR MATERIAL.pptx
ACCOUNTING FOR MATERIAL.pptx
 

Mais de Avinash Chavan

Report on Loss of Reading Habit Amoung Youth
Report on Loss of Reading Habit Amoung YouthReport on Loss of Reading Habit Amoung Youth
Report on Loss of Reading Habit Amoung YouthAvinash Chavan
 
Balance sheet And P & L as per Revised shedule six with exaple
Balance sheet And P & L as per Revised shedule six with exapleBalance sheet And P & L as per Revised shedule six with exaple
Balance sheet And P & L as per Revised shedule six with exapleAvinash Chavan
 
Balnce sheet And P & L as per Revised shedule six with exaple
Balnce sheet And P & L as per Revised shedule six with exapleBalnce sheet And P & L as per Revised shedule six with exaple
Balnce sheet And P & L as per Revised shedule six with exapleAvinash Chavan
 
Consolidated Financial Statements
Consolidated Financial StatementsConsolidated Financial Statements
Consolidated Financial StatementsAvinash Chavan
 
Turnaround Strategy & Case Study
Turnaround Strategy & Case Study Turnaround Strategy & Case Study
Turnaround Strategy & Case Study Avinash Chavan
 
Disaster Management Process with Case Study
Disaster Management Process with Case StudyDisaster Management Process with Case Study
Disaster Management Process with Case StudyAvinash Chavan
 
valuation of Goodwill and Share
valuation of Goodwill and Sharevaluation of Goodwill and Share
valuation of Goodwill and ShareAvinash Chavan
 
Bretton Woods system of monetary management
Bretton Woods system of monetary management Bretton Woods system of monetary management
Bretton Woods system of monetary management Avinash Chavan
 
Break-Even Analysis and Break-Even Point
Break-Even Analysis and Break-Even PointBreak-Even Analysis and Break-Even Point
Break-Even Analysis and Break-Even PointAvinash Chavan
 
Fiscal policies to correct disequilibrium in bop
Fiscal policies to correct disequilibrium in bopFiscal policies to correct disequilibrium in bop
Fiscal policies to correct disequilibrium in bopAvinash Chavan
 
FDI POLICY FRAME WORK IN INDIA
FDI POLICY FRAME WORK IN INDIAFDI POLICY FRAME WORK IN INDIA
FDI POLICY FRAME WORK IN INDIAAvinash Chavan
 
Techniques of overcoming resistance to change
Techniques of overcoming resistance to changeTechniques of overcoming resistance to change
Techniques of overcoming resistance to changeAvinash Chavan
 
Corporate social respponsibilty by max india foundation
Corporate social respponsibilty by max india foundationCorporate social respponsibilty by max india foundation
Corporate social respponsibilty by max india foundationAvinash Chavan
 
Disaster Mangement process with Uttarakhand Case Study
Disaster Mangement process with Uttarakhand Case StudyDisaster Mangement process with Uttarakhand Case Study
Disaster Mangement process with Uttarakhand Case StudyAvinash Chavan
 

Mais de Avinash Chavan (19)

Report on Loss of Reading Habit Amoung Youth
Report on Loss of Reading Habit Amoung YouthReport on Loss of Reading Habit Amoung Youth
Report on Loss of Reading Habit Amoung Youth
 
Balance sheet And P & L as per Revised shedule six with exaple
Balance sheet And P & L as per Revised shedule six with exapleBalance sheet And P & L as per Revised shedule six with exaple
Balance sheet And P & L as per Revised shedule six with exaple
 
Balnce sheet And P & L as per Revised shedule six with exaple
Balnce sheet And P & L as per Revised shedule six with exapleBalnce sheet And P & L as per Revised shedule six with exaple
Balnce sheet And P & L as per Revised shedule six with exaple
 
Turnaround strategy
Turnaround strategyTurnaround strategy
Turnaround strategy
 
Consolidated Financial Statements
Consolidated Financial StatementsConsolidated Financial Statements
Consolidated Financial Statements
 
Turnaround Strategy & Case Study
Turnaround Strategy & Case Study Turnaround Strategy & Case Study
Turnaround Strategy & Case Study
 
DISPUTES UNDER WTO
DISPUTES UNDER WTODISPUTES UNDER WTO
DISPUTES UNDER WTO
 
Disaster Management Process with Case Study
Disaster Management Process with Case StudyDisaster Management Process with Case Study
Disaster Management Process with Case Study
 
valuation of Goodwill and Share
valuation of Goodwill and Sharevaluation of Goodwill and Share
valuation of Goodwill and Share
 
Growth Strategy
 Growth Strategy Growth Strategy
Growth Strategy
 
Bretton Woods system of monetary management
Bretton Woods system of monetary management Bretton Woods system of monetary management
Bretton Woods system of monetary management
 
MASTER BUDGET
MASTER BUDGETMASTER BUDGET
MASTER BUDGET
 
Break-Even Analysis and Break-Even Point
Break-Even Analysis and Break-Even PointBreak-Even Analysis and Break-Even Point
Break-Even Analysis and Break-Even Point
 
valuation of Goodwill
valuation of Goodwillvaluation of Goodwill
valuation of Goodwill
 
Fiscal policies to correct disequilibrium in bop
Fiscal policies to correct disequilibrium in bopFiscal policies to correct disequilibrium in bop
Fiscal policies to correct disequilibrium in bop
 
FDI POLICY FRAME WORK IN INDIA
FDI POLICY FRAME WORK IN INDIAFDI POLICY FRAME WORK IN INDIA
FDI POLICY FRAME WORK IN INDIA
 
Techniques of overcoming resistance to change
Techniques of overcoming resistance to changeTechniques of overcoming resistance to change
Techniques of overcoming resistance to change
 
Corporate social respponsibilty by max india foundation
Corporate social respponsibilty by max india foundationCorporate social respponsibilty by max india foundation
Corporate social respponsibilty by max india foundation
 
Disaster Mangement process with Uttarakhand Case Study
Disaster Mangement process with Uttarakhand Case StudyDisaster Mangement process with Uttarakhand Case Study
Disaster Mangement process with Uttarakhand Case Study
 

Último

How to do quick user assign in kanban in Odoo 17 ERP
How to do quick user assign in kanban in Odoo 17 ERPHow to do quick user assign in kanban in Odoo 17 ERP
How to do quick user assign in kanban in Odoo 17 ERPCeline George
 
Procuring digital preservation CAN be quick and painless with our new dynamic...
Procuring digital preservation CAN be quick and painless with our new dynamic...Procuring digital preservation CAN be quick and painless with our new dynamic...
Procuring digital preservation CAN be quick and painless with our new dynamic...Jisc
 
ACC 2024 Chronicles. Cardiology. Exam.pdf
ACC 2024 Chronicles. Cardiology. Exam.pdfACC 2024 Chronicles. Cardiology. Exam.pdf
ACC 2024 Chronicles. Cardiology. Exam.pdfSpandanaRallapalli
 
Global Lehigh Strategic Initiatives (without descriptions)
Global Lehigh Strategic Initiatives (without descriptions)Global Lehigh Strategic Initiatives (without descriptions)
Global Lehigh Strategic Initiatives (without descriptions)cama23
 
Barangay Council for the Protection of Children (BCPC) Orientation.pptx
Barangay Council for the Protection of Children (BCPC) Orientation.pptxBarangay Council for the Protection of Children (BCPC) Orientation.pptx
Barangay Council for the Protection of Children (BCPC) Orientation.pptxCarlos105
 
Karra SKD Conference Presentation Revised.pptx
Karra SKD Conference Presentation Revised.pptxKarra SKD Conference Presentation Revised.pptx
Karra SKD Conference Presentation Revised.pptxAshokKarra1
 
USPS® Forced Meter Migration - How to Know if Your Postage Meter Will Soon be...
USPS® Forced Meter Migration - How to Know if Your Postage Meter Will Soon be...USPS® Forced Meter Migration - How to Know if Your Postage Meter Will Soon be...
USPS® Forced Meter Migration - How to Know if Your Postage Meter Will Soon be...Postal Advocate Inc.
 
4.18.24 Movement Legacies, Reflection, and Review.pptx
4.18.24 Movement Legacies, Reflection, and Review.pptx4.18.24 Movement Legacies, Reflection, and Review.pptx
4.18.24 Movement Legacies, Reflection, and Review.pptxmary850239
 
4.16.24 21st Century Movements for Black Lives.pptx
4.16.24 21st Century Movements for Black Lives.pptx4.16.24 21st Century Movements for Black Lives.pptx
4.16.24 21st Century Movements for Black Lives.pptxmary850239
 
INTRODUCTION TO CATHOLIC CHRISTOLOGY.pptx
INTRODUCTION TO CATHOLIC CHRISTOLOGY.pptxINTRODUCTION TO CATHOLIC CHRISTOLOGY.pptx
INTRODUCTION TO CATHOLIC CHRISTOLOGY.pptxHumphrey A Beña
 
Inclusivity Essentials_ Creating Accessible Websites for Nonprofits .pdf
Inclusivity Essentials_ Creating Accessible Websites for Nonprofits .pdfInclusivity Essentials_ Creating Accessible Websites for Nonprofits .pdf
Inclusivity Essentials_ Creating Accessible Websites for Nonprofits .pdfTechSoup
 
Virtual-Orientation-on-the-Administration-of-NATG12-NATG6-and-ELLNA.pdf
Virtual-Orientation-on-the-Administration-of-NATG12-NATG6-and-ELLNA.pdfVirtual-Orientation-on-the-Administration-of-NATG12-NATG6-and-ELLNA.pdf
Virtual-Orientation-on-the-Administration-of-NATG12-NATG6-and-ELLNA.pdfErwinPantujan2
 
What is Model Inheritance in Odoo 17 ERP
What is Model Inheritance in Odoo 17 ERPWhat is Model Inheritance in Odoo 17 ERP
What is Model Inheritance in Odoo 17 ERPCeline George
 
Choosing the Right CBSE School A Comprehensive Guide for Parents
Choosing the Right CBSE School A Comprehensive Guide for ParentsChoosing the Right CBSE School A Comprehensive Guide for Parents
Choosing the Right CBSE School A Comprehensive Guide for Parentsnavabharathschool99
 
Judging the Relevance and worth of ideas part 2.pptx
Judging the Relevance  and worth of ideas part 2.pptxJudging the Relevance  and worth of ideas part 2.pptx
Judging the Relevance and worth of ideas part 2.pptxSherlyMaeNeri
 
Culture Uniformity or Diversity IN SOCIOLOGY.pptx
Culture Uniformity or Diversity IN SOCIOLOGY.pptxCulture Uniformity or Diversity IN SOCIOLOGY.pptx
Culture Uniformity or Diversity IN SOCIOLOGY.pptxPoojaSen20
 
AUDIENCE THEORY -CULTIVATION THEORY - GERBNER.pptx
AUDIENCE THEORY -CULTIVATION THEORY -  GERBNER.pptxAUDIENCE THEORY -CULTIVATION THEORY -  GERBNER.pptx
AUDIENCE THEORY -CULTIVATION THEORY - GERBNER.pptxiammrhaywood
 

Último (20)

How to do quick user assign in kanban in Odoo 17 ERP
How to do quick user assign in kanban in Odoo 17 ERPHow to do quick user assign in kanban in Odoo 17 ERP
How to do quick user assign in kanban in Odoo 17 ERP
 
LEFT_ON_C'N_ PRELIMS_EL_DORADO_2024.pptx
LEFT_ON_C'N_ PRELIMS_EL_DORADO_2024.pptxLEFT_ON_C'N_ PRELIMS_EL_DORADO_2024.pptx
LEFT_ON_C'N_ PRELIMS_EL_DORADO_2024.pptx
 
Procuring digital preservation CAN be quick and painless with our new dynamic...
Procuring digital preservation CAN be quick and painless with our new dynamic...Procuring digital preservation CAN be quick and painless with our new dynamic...
Procuring digital preservation CAN be quick and painless with our new dynamic...
 
ACC 2024 Chronicles. Cardiology. Exam.pdf
ACC 2024 Chronicles. Cardiology. Exam.pdfACC 2024 Chronicles. Cardiology. Exam.pdf
ACC 2024 Chronicles. Cardiology. Exam.pdf
 
Global Lehigh Strategic Initiatives (without descriptions)
Global Lehigh Strategic Initiatives (without descriptions)Global Lehigh Strategic Initiatives (without descriptions)
Global Lehigh Strategic Initiatives (without descriptions)
 
Barangay Council for the Protection of Children (BCPC) Orientation.pptx
Barangay Council for the Protection of Children (BCPC) Orientation.pptxBarangay Council for the Protection of Children (BCPC) Orientation.pptx
Barangay Council for the Protection of Children (BCPC) Orientation.pptx
 
Karra SKD Conference Presentation Revised.pptx
Karra SKD Conference Presentation Revised.pptxKarra SKD Conference Presentation Revised.pptx
Karra SKD Conference Presentation Revised.pptx
 
USPS® Forced Meter Migration - How to Know if Your Postage Meter Will Soon be...
USPS® Forced Meter Migration - How to Know if Your Postage Meter Will Soon be...USPS® Forced Meter Migration - How to Know if Your Postage Meter Will Soon be...
USPS® Forced Meter Migration - How to Know if Your Postage Meter Will Soon be...
 
4.18.24 Movement Legacies, Reflection, and Review.pptx
4.18.24 Movement Legacies, Reflection, and Review.pptx4.18.24 Movement Legacies, Reflection, and Review.pptx
4.18.24 Movement Legacies, Reflection, and Review.pptx
 
4.16.24 21st Century Movements for Black Lives.pptx
4.16.24 21st Century Movements for Black Lives.pptx4.16.24 21st Century Movements for Black Lives.pptx
4.16.24 21st Century Movements for Black Lives.pptx
 
INTRODUCTION TO CATHOLIC CHRISTOLOGY.pptx
INTRODUCTION TO CATHOLIC CHRISTOLOGY.pptxINTRODUCTION TO CATHOLIC CHRISTOLOGY.pptx
INTRODUCTION TO CATHOLIC CHRISTOLOGY.pptx
 
Inclusivity Essentials_ Creating Accessible Websites for Nonprofits .pdf
Inclusivity Essentials_ Creating Accessible Websites for Nonprofits .pdfInclusivity Essentials_ Creating Accessible Websites for Nonprofits .pdf
Inclusivity Essentials_ Creating Accessible Websites for Nonprofits .pdf
 
Virtual-Orientation-on-the-Administration-of-NATG12-NATG6-and-ELLNA.pdf
Virtual-Orientation-on-the-Administration-of-NATG12-NATG6-and-ELLNA.pdfVirtual-Orientation-on-the-Administration-of-NATG12-NATG6-and-ELLNA.pdf
Virtual-Orientation-on-the-Administration-of-NATG12-NATG6-and-ELLNA.pdf
 
What is Model Inheritance in Odoo 17 ERP
What is Model Inheritance in Odoo 17 ERPWhat is Model Inheritance in Odoo 17 ERP
What is Model Inheritance in Odoo 17 ERP
 
Choosing the Right CBSE School A Comprehensive Guide for Parents
Choosing the Right CBSE School A Comprehensive Guide for ParentsChoosing the Right CBSE School A Comprehensive Guide for Parents
Choosing the Right CBSE School A Comprehensive Guide for Parents
 
YOUVE_GOT_EMAIL_PRELIMS_EL_DORADO_2024.pptx
YOUVE_GOT_EMAIL_PRELIMS_EL_DORADO_2024.pptxYOUVE_GOT_EMAIL_PRELIMS_EL_DORADO_2024.pptx
YOUVE_GOT_EMAIL_PRELIMS_EL_DORADO_2024.pptx
 
Judging the Relevance and worth of ideas part 2.pptx
Judging the Relevance  and worth of ideas part 2.pptxJudging the Relevance  and worth of ideas part 2.pptx
Judging the Relevance and worth of ideas part 2.pptx
 
Culture Uniformity or Diversity IN SOCIOLOGY.pptx
Culture Uniformity or Diversity IN SOCIOLOGY.pptxCulture Uniformity or Diversity IN SOCIOLOGY.pptx
Culture Uniformity or Diversity IN SOCIOLOGY.pptx
 
FINALS_OF_LEFT_ON_C'N_EL_DORADO_2024.pptx
FINALS_OF_LEFT_ON_C'N_EL_DORADO_2024.pptxFINALS_OF_LEFT_ON_C'N_EL_DORADO_2024.pptx
FINALS_OF_LEFT_ON_C'N_EL_DORADO_2024.pptx
 
AUDIENCE THEORY -CULTIVATION THEORY - GERBNER.pptx
AUDIENCE THEORY -CULTIVATION THEORY -  GERBNER.pptxAUDIENCE THEORY -CULTIVATION THEORY -  GERBNER.pptx
AUDIENCE THEORY -CULTIVATION THEORY - GERBNER.pptx
 

STANDARD COSTING-DETAILED ANALYSIS OF MATERIAL VARIANCE

  • 1. 1 STANDARD COSTING-DETAILED ANALYSIS OF MATERIAL VARIANVE Introduction to Standard Costing Standard costing is an important subtopic of cost accounting. Standard costs are usually associated with a manufacturing company's costs of direct material, direct labor, and manufacturing overhead. Rather than assigning the actual costs of direct material, direct labor, and manufacturing overhead to a product, many manufacturers assign the expected or standard cost. This means that a manufacturer's inventories and cost of goods sold will begin with amounts reflecting the standard costs, not the actual costs, of a product. Manufacturers, of course, still have to pay the actual costs. As a result there are almost always differences between the actual costs and the standard costs, and those differences are known as variances. Standard costing and the related variances is a valuable management tool. If a variance arises, management becomes aware that manufacturing costs have differed from the standard (planned, expected) costs. If actual costs are greater than standard costs the variance is unfavorable. An unfavorable variance tells management that if everything else stays constant the company's actual profit will be less than planned. If actual costs are less than standard costs the variance is favorable. A favorable variance tells management that if everything else stays constant the actual profit will likely exceed the planned profit. The sooner that the accounting system reports a variance, the sooner that management can direct its attention to the difference from the planned amounts. If we assume that a company uses the perpetual inventory system and that it carries all of its inventory accounts at standard cost (including Direct Materials Inventory or Stores), then the standard cost of a finished product is the sum of the standard costs of the inputs:
  • 2. 2 Direct material Direct labor Manufacturing overhead Variable manufacturing overhead Fixed manufacturing overhead Usually there will be two variances computed for each input: Since the calculation of variances can be difficult, we developed several business forms to help you get started and to understand what the variances tell us. Let's assume that your Uncle Pete runs a retail outlet that sells denim aprons in two sizes. Pete suggests that you get into the manufacturing side of the business, so on January 1, 2013 you start up an apron production company called Denim Works. Using the best information at hand, the two of you compile the following estimates to use as standards for 2013: The aprons are easy to produce, and no apron is ever left unfinished at the end of any given day. This means that your company never has work-in-process inventory. When we make your journal entries for completed aprons (shown below), we'll use an account called Inventory-FG which means Finished Goods Inventory. (Some companies will use WIP Inventory or Work-in-Process Inventory). We'll also use the account Direct Materials Inventory. (Other account titles often used for direct materials are Raw Materials Inventory or Stores.) Direct Materials Purchased: Standard Cost and Price Variance Direct materials refer to just that—raw materials that are directly traceable into a product. In your apron business the direct material is the denim. (In a food manufacturer's business the direct materials are the ingredients such as flour and sugar; in an automobile assembly plant, the direct materials are the cars' component parts). Denim Works purchases its denim from a local supplier with terms of net 30 days, FOB destination. This means that title to the denim passes from the supplier to Denim Works when
  • 3. 3 Denim Works receives the material. When the denim arrives, Denim Works will record the denim received in its Direct Materials Inventory at the standard cost of rs3 per yard (see standards table above) and will record the liability at the actual cost for the amount received. Any difference between the standard cost of the material and the actual cost of the material received is recorded as a purchase price variance. Examples of Standard Cost of Materials and Price Variance Let's assume that on January 2, 2013 Denim Works ordered 1,000 yards of denim at rs2.90 per yard. On January 8, 2013 Denim Works receives 1,000 yards of denim and an invoice for the actual cost of rs2,900. On January 8, 2013 Denim Works becomes the owner of the material and has a liability to its supplier. On January 8 Denim Works' Direct Materials Inventory is increased by the standard cost of rs3,000 (1,000 yards of denim at the standard cost of rs3 per yard), Accounts Payable is credited for rs2,900 (the actual amount owed to the supplier), and the difference of rs100 is credited to Direct Materials Price Variance. In general journal format the entry looks like this: The rs100 credit to the price variance account communicates immediately (when the denim arrives) that the company is experiencing actual costs that are more favorable than the planned, standard cost. In February, Denim Works orders 3,000 yards of denim at rs3.05 per yard. On March 1, 2013 Denim Works receives the 3,000 yards of denim and an invoice for rs9,150 due in 30 days. On March 1, the Direct Materials Inventory account is increased by the standard cost of rs9,000 (3,000 yards at the standard cost of rs3 per yard), Accounts Payable is credited for rs9,150 (the actual cost of the denim), and the difference of rs150 is debited to Direct Materials Price Variance as an unfavorable price variance: After the March 1 transaction is posted, the Direct Materials Price Variance account shows a debit balance of rs50 (the rs100 credit on January 2 combined with the rs150 debit on March 1). A debit balance in a variance account is always unfavorable—it shows that the total of actual costs is higher than the total of the expected standard costs. In other words, your company's profit will be rs50 less than planned unless you take some action.
  • 4. 4 On June 1 your company receives 3,000 yards of denim at an actual cost of rs2.92 per yard for a total of rs8,760 due in 30 days. The entry is: Direct Materials Inventory is debited for the standard cost of rs9,000 (3,000 yards at rs3 per yard), Accounts Payable is credited for the actual amount owed, and the difference of rs240 is credited to Direct Materials Price Variance. A credit to the variance account indicates that the actual cost is less than the standard cost. After this transaction is recorded, the Direct Materials Price Variance account shows an overall credit balance of rs190. A credit balance in a variance account is always favorable. In other words, your company's profit will be rs190 greater than planned due to the favorable cost of direct materials. Note that the entire price variance pertaining to all of the direct materials received was recorded immediately. In other words, the price variance associated with the direct materials received was not delayed until the materials were used. We will discuss later how to handle the balances in the variance accounts under the heading "What To Do With Variance Amounts". Direct Materials Usage Variance Under a standard costing system, production and inventories are reported at the standard cost— including the standard quantity of direct materials that should have been used to make the products. If the manufacturer actually uses more direct materials than the standard quantity of materials for the products actually manufactured, the company will have an unfavorable direct materials usage variance. If the quantity of direct materials actually used is less than the standard quantity for the products produced, the company will have a favorable usage variance. The amount of a favorable and unfavorable variance is recorded in a general ledger account Direct Materials Usage Variance. (Alternative account titles include Direct Materials Quantity Variance or Direct Materials Efficiency Variance.) Let's demonstrate this variance with the following information.
  • 5. 5 January 2013 In order to calculate the direct materials usage or quantity variance, we start with the number of acceptable units of products that have been manufactured—also known as the good output. At Denim Works this is the number of good aprons physically produced. If Denim Works produces 100 large aprons and 60 small aprons during January, the production and the finished goods inventory will begin with the cost of the direct materials that should have been used to make those aprons. Any difference will be a variance. Note: We are not determining the quantity of aprons that Denim Works should have made. Rather, we are determining whether the 100 large aprons and 60 small aprons that were actually manufactured were produced efficiently. In the case of direct materials, we want to determine whether or not the company used the proper amount of denim to make the 160 aprons that were actually produced. (For the purposes of calculating the direct materials usage variance, it does not concern us whether Denim Works had a goal to produce 100 aprons, 200, aprons, or 250 aprons.) Standard costs are sometimes referred to as the "should be costs." Denim Works should be using 278 yards of denim to make 100 large aprons and 60 small aprons as shown in the following table. We determine the total standard cost of the denim that should have been used to make the 160 aprons by multiplying the standard quantity of denim (278 yards) by the standard cost of a yard of denim (rs3 per yard):
  • 6. 6 An inventory account (such as F.G. Inventory or Work-in-Process) is debited for rs834; this is the standard cost of the direct materials component in the aprons manufactured in January 2013. The Direct Materials Inventory account is reduced by the standard cost of the denim actually removed from the direct materials inventory. Let's assume that the actual quantity of denim removed from the direct materials inventory and used to make the aprons in January was 290 yards. Because Direct Materials Inventory reports the standard cost of the actual materials on hand, we reduce the account balance by rs870 (290 yards used rs3 standard cost per yard). After removing 290 yards of materials, the balance in the Direct Materials Inventory account is rs2,130 (710 yards x rs3 standard cost per yard). The Direct Materials Usage Variance is: [the standard quantity of material that should have been used to make the good output minus the actual quantity of material used] X the standard cost per yard. In our example, Denim Works should have used 278 yards of material to make 100 large aprons and 60 small aprons. Because the company actually used 290 yards of denim, we say that Denim Works did not operate efficiently—an extra 12 yards of denim was used (278 vs. 290 = 12). When we multiply the 12 yards by the standard cost of rs3 per yard, the result is an unfavorable direct materials usage variance of rs36. Let's put the above information into a format commonly used for computing variances:
  • 7. 7 Direct Materials Usage/Quantity/Efficiency Variance Analysis The journal entry for the direct materials portion of the January production is: February 2013 Let's assume that in February 2013 Denim Works produces 200 large aprons and 100 small aprons and that 520 yards of denim are actually used. From this information we can compute the following:
  • 8. 8 Let's put the above information into our format: Direct Materials Usage (or Quantity) Variance Analysis The journal entry for the direct materials portion of the February production is:
  • 9. 9 Standard Costing Standard costing is the practice of substituting an expected cost for an actual cost in the accounting records, and then periodically recording variances that are the difference between the expected and actual costs. This approach represents a simplified alternative to cost layering systems, such as the FIFO and LIFO methods, where large amounts of historical cost information must be maintained for items held in stock. Standard costing involves the creation of estimated (i.e., standard) costs for some or all activities within a company. The core reason for using standard costs is that there are a number of applications where it is too time-consuming to collect actual costs, so standard costs are used as a close approximation to actual costs. Since standard costs are usually slightly different from actual costs, the cost accountant periodically calculates variances that break out differences caused by such factors as labor rate changes and the cost of materials. The cost accountant may also periodically change the standard costs to bring them into closer alignment with actual costs. Advantages of Standard Costing Though most companies do not use standard costing in its original application of calculating the cost of ending inventory, it is still useful for a number of other applications. In most cases, users are probably not even aware that they are using standard costing, only that they are using an approximation of actual costs. Here are some potential uses: Budgeting: A budget is always composed of standard costs, since it would be impossible to include in it the exact actual cost of an item on the day the budget is finalized. Also, since a key application of the budget is to compare it to actual results in subsequent periods, the standards used within it continue to appear in financial reports through the budget period. Inventory costing: It is extremely easy to print a report showing the period-end inventory balances (if you are using a perpetual inventory system), multiply it by the standard cost of each item, and instantly generate an ending inventory valuation. The result does not exactly match the actual cost of inventory, but it is close. However, it may be necessary to update standard costs frequently, if actual costs are continually changing. It is easiest to update costs for the highest-
  • 10. 10 dollar components of inventory on a frequent basis, and leave lower-value items for occasional cost reviews. Overhead application: If it takes too long to aggregate actual costs into cost pools for allocation to inventory, then you may use a standard overhead application rate instead, and adjust this rate every few months to keep it close to actual costs. Price formulation: If a company deals with custom products, then it uses standard costs to compile the projected cost of a customer’s requirements, after which it adds on a margin. This may be quite a complex system, where the sales department uses a database of component costs that change depending upon the unit quantity that the customer wants to order. This system may also account for changes in the company’s production costs at different volume levels, since this may call for the use of longer production runs that are less expensive. Nearly all companies have budgets and many use standard cost calculations to derive product prices, so it is apparent that standard costing will find some uses for the foreseeable future. In particular, standard costing provides a benchmark against which management can compare actual performance. Problems with Standard Costing Despite the advantages just noted for some applications of standard costing, there are substantially more situations where it is not a viable costing system. Here are some problem areas: Cost-plus contracts: If you have a contract with a customer under which the customer pays you for your costs incurred, plus a profit (known as a cost-plus contract), then you must use actual costs, as per the terms of the contract. Standard costing is not allowed. Drives inappropriate activities: A number of the variances reported under a standard costing system will drive management to take incorrect actions to create favorable variances. For example, they may buy raw materials in larger quantities in order to improve the purchase price variance, even though this increases the investment in inventory. Similarly, management may schedule longer production runs in order to improve the labor efficiency variance, even though it is better to produce in smaller quantities and accept less labor efficiency in exchange.
  • 11. 11 Fast-paced environment: A standard costing system assumes that costs do not change much in the near term, so that you can rely on standards for a number of months or even a year, before updating the costs. However, in an environment where product lives are short or continuous improvement is driving down costs, a standard cost may become out-of-date within a month or two. Slow feedback: A complex system of variance calculations are an integral part of a standard costing system, which the accounting staff completes at the end of each reporting period. If the production department is focused on immediate feedback of problems for instant correction, the reporting of these variances is much too late to be useful. Unit-level information: The variance calculations that typically accompany a standard costing report are accumulated in aggregate for a company’s entire production department, and so are unable to provide information about discrepancies at a lower level, such as the individual work cell, batch, or unit. The preceding list shows that there are a multitude of situations where standard costing is not useful, and may even result in incorrect management actions. Nonetheless, as long as you are aware of these issues, it is usually possible to profitably adapt standard costing into some aspects of a company’s operations. Standard Cost Variances A variance is the difference between the actual cost incurred and the standard cost against which it is measured. A variance can also be used to measure the difference between actual and expected sales. Thus, variance analysis can be used to review the performance of both revenue and expenses. There are two basic types of variances from a standard that can arise, which are the rate variance and the volume variance. Here is more information about both types of variances: Rate variance: A rate variance (which is also known as a price variance) is the difference between the actual price paid for something and the expected price, multiplied by the actual quantity purchased. The “rate” variance designation is most commonly applied to the labor rate variance, which involves the actual cost of direct labor in comparison to the standard cost of
  • 12. 12 direct labor. The rate variance uses a different designation when applied to the purchase of materials, and may be called the purchase price variance or the material price variance. Volume variance: A volume variance is the difference between the actual quantity sold or consumed and the budgeted amount, multiplied by the standard price or cost per unit. If the variance relates to the sale of goods, it is called the sales volume variance. If it relates to the use of direct materials, it is called the material yield variance. If the variance relates to the use of direct labor, it is called the labor efficiency variance. Finally, if the variance relates to the application of overhead, it is called the overhead efficiency variance. Thus, variances are based on either changes in cost from the expected amount, or changes in the quantity from the expected amount. The most common variances that a cost accountant elects to report on are subdivided within the rate and volume variance categories for direct materials, direct labor, and overhead. It is also possible to report these variances for revenue. It is not always considered practical or even necessary to calculate and report on variances, unless the resulting information can be used by management to improve the operations or lower the costs of a business. When a variance is considered to have a practical application, the cost accountant should research the reason for the variance in considerable detail and present the results to the responsible manager, perhaps also with a suggested course of action. Standard Cost Creation At the most basic level, you can create a standard cost simply by calculating the average of the most recent actual cost for the past few months. In many smaller companies, this is the extent of the analysis used. However, there are some additional factors to consider, which can significantly alter the standard cost that you elect to use. They are: Equipment age: If a machine is nearing the end of its productive life, it may produce a higher proportion of scrap than was previously the case. Equipment setup speeds: If it takes a long time to setup equipment for a production run, the cost of the setup, as spread over the units in the production run, is expensive. If a setup reduction plan is contemplated, this can yield significantly lower overhead costs.
  • 13. 13 Labor efficiency changes: If there are production process changes, such as the installation of new, automated equipment, then this impact the amount of labor required to manufacture a product. Labor rate changes: If you know that employees are about to receive pay raises, either through a scheduled raise or as mandated by a labor union contract, then incorporate it into the new standard. This may mean setting an effective date for the new standard that matches the date when the cost increase is supposed to go into effect. Learning curve: As the production staff creates an increasing volume of a product, it becomes more efficient at doing so. Thus, the standard labor cost should decrease (though at a declining rate) as production volumes increase. Purchasing terms: The purchasing department may be able to significantly alter the price of a purchased component by switching suppliers, altering contract terms, or by buying in different quantities. Any one of the additional factors noted here can have a major impact on a standard cost, which is why it may be necessary in a larger production environment to spend a significant amount of time formulating a standard cost.
  • 14. 14 FORMULA Direct Materials Variances: Materials purchase price variance Formula: Materials purchase price variance = (Actual quantity purchased × Actual price) – (Actual quantity purchased × Standard price) Materials price usage variance formula Materials price usage variance = (Actual quantity used × Actual price) – (Actual quantity used × Standard price) materials quantity / usage variance formula Materials price usage variance = (Actual quantity used × Standard price) – (Standard quantity allowed × Standard price) Materials mix variance formula (Actual quantities at individual standard materials costs) – (Actual quantities at weighted average of standard materials costs) Materials yield variance formula (Actual quantities at weighted average of standard materials costs) – (Actual output quantity at standard materials cost)
  • 15. 15 MATERIAL COST VARIANCE (SQ*SP) - (AQ*AP) STD COST – ACT COST MATERIAL USAGE VARIANCE (SQ*SP) - (AQ*SP) (SQ-AQ) * SP MATERIAL PRICE VARIANCE (SP*AQ) - (AP*AQ) (SP-AP) *AQ MATERIAL MIX VARIANCE 𝑇𝑄 𝑜𝑓 𝑎𝑐𝑡𝑢𝑎𝑙 𝑚𝑖𝑥 𝑇𝑄 𝑜𝑓 𝑠𝑡𝑎𝑛𝑑𝑎𝑟𝑑 𝑚𝑖𝑥 ∗ 𝑠𝑡𝑎𝑛𝑑𝑎𝑟𝑑 𝑄 𝑜𝑓 𝑥 MATERIAL YEILD VARIANCE (SP*SQ) – (SP*RQ) (SQ-RQ) *SP I.e. MCV = MUV + MPV MCV = MMV + MYV
  • 16. 16 Materialcostvariance Learning Objectives To know how to calculate material cost variance To know how to calculate material usage variance To know how to calculate material price variance Terminology for Material Variance Calculation SQ =Standard Quantity SR = Standard Rate AQ = Actual Quantity AP = Actual Price RSQ = Resaved STD Quantity MCV = Material Cost Variance MUV = Material Usage Variance MPV = Material Price Variance MMV = Material Mix Variance MYV = Material Yield Variance
  • 17. 17 Problem 1 The following information is obtained from X Co. Ltd.Product SQ SP AQ AP(Units) (Rs.) (Units) (Rs.)A 1050 2.00 1100 2.25B 1500 3.25 1400 3. 50C 2100 3.50 2000 3.75Calculate the following: Material cost variance Material price variance Material usage variance Solution 1. MCV = (SQ x SR) – (AQ x AR)A = (1050 x 2.00) – (1100 x 2.25)= 2100 – 2475 = Rs. 375 (A)B = (1500 x 3.25) – (1400 x 3.50)= 4875 – 4900 = 25 (A) 2C = (2100 x 3.50) – (2000 x 7.25)= 7350 – 7500 = 150 (A)--------------------550 A---------------- ----2. MPV = AQ (SP-AP)A = 1100(2.00 – 2.25) = 1100 x (0.25) = Rs. 275 (A)B = 1400(3.25 – 3.50) = 1400 x (-0.25) = Rs. 350 (A)C = 2000(3.50 – 3.75) = 2000 x (-0.25) = Rs. 500 (A)------- ----------1125 A-----------------3. MUV = SP (SQ-AQ)A = 2 (1050 -1100) = 2 (- 50) = Rs. 100 (A)B = 3.25 (1500 – 1400) = 3.25 (100) = Rs. 325 (F)C = 3.50 (2100 - 2000) = 3.50 (100) = Rs. 350 (F)---------------575 (F)--------------- Verification MCV = MPV + MUVRs. 550 (A) = 1125(A) + 575 F550 (A) = 550 (A) Problem 2 Product SQ SP Total AQ AP TotalA 10 2 20 5 3 15B 20 3 60 10 6 60C 20 6 120 15 5 75--- ----- - -- -----Total 50 200 30 150--- ----- --- -----Find out (1) MCV (2) MPV (3) MMV (4) MUV Sol.: (1) MCV = (SQ x SR) – (AQ x AR)A = (10 x 2) – (5 x 3) = 5 FB = (20 x 3) – (10 x 6) = Nil
  • 18. 18 Direct Material Price Variance Direct material price variance (also called the direct material spending/rate variance) is the product of actual quantity of direct material used and the difference between standard price and actual price per unit of direct material. It is calculated using the following formula: DM Price Variance = (SP − AP) × AQ Where, SP is the standard unit price of direct material AP is the actual price per unit of direct material AQ is the actual quantity of direct material used Analysis Direct material price variance is calculated to determine the efficiency of purchasing department in obtaining direct material at low cost. A positive value of direct material price variance is favorable which means that direct material was purchased for lesser amount than the standard price. A negative value of direct material price variance is unfavorable because more than estimated price per unit is paid. However, a favorable direct material price variance is not always good; it should be analyzed together with direct material quantity variance. It is quite possible that the purchasing department may purchase low quality raw material to generate a favorable direct material price variance. Such a favorable material price variance will be offset by an unfavorable direct material quantity variance due to wastage of low quality direct material. Example Calculate the direct material price variance if the standard price and actual unit price per unit of direct material are RS4.00 and RS4.10 respectively; and actual units of direct material used during the period are 1,200. Determine whether the variance is favorable or unfavorable.
  • 19. 19 Standard Price RS4.00 − Actual Price 4.10 Difference Per Unit − 0.10 × Actual Quantity 1,200 Direct Material Price Variance − RS 120 Since the price paid by the company for the purchase of direct material is more than the standard price by RS120, the DM price variance is unfavorable. In managerial accounting, variance means deviation of actual costs from standard costs. Materials price variance is the result of deviation of actual price paid for materials from what has been set as standard. Direct materials price and quantity standards are set after keeping in mind the current market prices and anticipated changes in materials prices in near future. However things do not always happen as expected. The actual price of materials may significantly deviate from standard price. Moreover, the expenses associated with the order (freight, duties, handling expenses etc.) may increase or reduce the price of materials available for use. The business may have to pay more or less price than what has been considered as normal at the time of setting standards. If the actual price paid for materials is more than the standard price, an unfavorable materials price variance occurs. On the other hand, if the actual price paid for the materials is less than the standard price, a favorable materials price variance occurs. An example is given below for more explanation: Example The Aptex Company manufactures and sells small speakers that are used in mobile phones. The speakers are sold in bulk to mobile manufacturing companies where complete mobiles are produced. The direct material of Aptex Company is a thin copper coil. One meter of the copper coil is the standard requirement to manufacture one speaker. The standard cost to manufacture one speaker is as follows:
  • 20. 20 Direct materials: 1meters × RS1.50 per meters RS1.50 Direct labor RS1.00 Manufacturing overhead RS0.50 —– Total RS3.00 —– During the month of June, Aptex purchased 5,000 meters of copper coil @ RS1.70 per meter and produced 2,500 speakers using 3,000 meters of copper coil. The materials price variance for the month of June can be calculated using the following formula/equation: = (5,000 × RS1.70) – (5,000 × RS1.50) = RS8,500 – RS7,500 = 1,000 Unfavorable Aptex has an unfavorable materials price variance for June because the actual price paid (RS8,500) is more than the standard price allowed (RS7,500) for 5,000 meters of copper coil. This variance can also be computed by using the factored form of above formula: AQ × (AP – SP) 5,000 meters × (RS1.70 – RS1.50) 5,000 meters × RS0.20 RS1,000 Unfavorable
  • 21. 21 Reasons of direct materials price variance: A favorable or unfavorable material price variance may occur due to one or more of the following reasons: Order size: Some suppliers allow discount on large orders. The materials purchased in large quantities may reduce the unit price and a favorable price variance may occur. Rise in price: The rise in the general price level may increase the input costs of the vendor and as a result vendor may increase the price of the materials. The rise in price is very common reason of an unfavorable variance. Urgent needs: If production department does not indicate the need of materials on time, the purchasing department may have to order on urgent basis that may increase the price of materials and other expenses associated with the order. Quality: A favorable price variance may be the result of purchasing low quality materials and an unfavorable variance may be the result of purchasing high quality materials. Inefficient standard setting: Inefficiencies in terms of forecasting and environmental scanning during standard setting process can be a reason of huge variances. Transportation: Transportation is a part of total direct materials cost. Any change in the transportation expenses can change the total and per unit cost of direct materials available for use and can become the reason of favorable or unfavorable direct materials price variance. The role of just in time manufacturing: A company that operates under just in time (JIT) manufacturing system may have to face shortage of direct materials due to a sudden increase in demand for the product. The orders in rush normally increase the costs. In that case company will have to either accept an unfavorable materials price variance or lost sales. Inefficient or unreliable Suppliers: A deviation from standard material costs may be the result of inefficient or unreliable vendors. For example, if suppliers of raw materials are unable to meet the demand, the company may have to look for another supplier who may be more costly.
  • 22. 22 Responsibility of the variance: Purchasing department is responsible to place orders for direct materials so this variance is generally considered the responsibility of purchase manager. However, the above reasons clarify that the materials price variance may or may not be the result of inefficiencies of the purchasing department. The occurrence of variances is very normal. They occur for almost all cost elements and should not be used to find someone to blame. Sometimes they may not be very significant and sometimes they may be the result of the factors that are beyond the control of managers. Variances are tools to control costs, improve efficiencies and should be used positively. Direct Material Quantity Variance Direct material quantity variance (also called the direct material usage/efficiency variance) is the product of standard price of a unit of direct material and the difference between standard quantity of direct material allowed and actual quantity of direct material used. The formula to calculate direct material quantity variance is: DM Quantity Variance = ( SQ − AQ ) × SP Where, SQ is the standard quantity allowed AQ is the actual quantity of direct material used SP is the standard price per unit of direct material Standard quantity allowed (SQ) is calculated as the product of standard quantity of direct material per unit and actual units produced. Analysis Direct material quantity variance is calculated to determine the efficiency of production department in converting raw material to finished goods. In order to improve efficiency, wastage of raw material must be reduced. A negative value of direct material quantity variance is
  • 23. 23 unfavorable and it implies that more quantity of direct material has been used in the production process than actually needed. A positive value of direct material quantity variance is favorable implying that raw material was efficiently converted to finished goods. Example Use the following information to calculate direct material quantity variance. Also specify whether the variance is favorable or unfavorable. Standard Price of a Unit of Direct Material RS4 Standard Quantity of Direct Material Per Unit 2 Actual Units Produced During the Period 620 Actual Quantity Used During the Period 1,200 Solution Actual Units Produced 620 × Standard Quantity of Direct Material Per Unit 2 Standard Quantity Allowed 1,240 Standard Quantity Allowed 1,240 − Actual Quantity 1,200 Difference 40 × Standard Price of a Unit of Direct Material RS4 Direct Material Quantity Variance RS160 In this case the production department performed efficiently and saved 40 units of direct material. Multiplying this by standard price per unit yields a favorable direct material quantity variance of RS160.
  • 24. 24 Direct Material Usage Variance Direct Material Usage Variance is the measure of difference between the actual quantity of material utilized during a period and the standard consumption of material for the level of output achieved. Direct Material Usage Variance: = Actual Quantity x Standard Price - Standard Quantity x Standard Price = Standard Cost of Actual Quantity - Standard Cost of Standard Quantity = (Actual Quantity - Standard Quantity) x Standard Price Since the effect of any variation in material price from the standard is calculated in the material price variance, material usage variance is calculated using the standard price. Example Cement PLC manufactured 10,000 bags of cement during the month of January. Consumption of raw materials during the period was as follows: Material Quantity Used Standard Usage Per Bag Actual Price Standard Price Limestone 100 tons 11 KG RS75/ton RS70/ton Clay 150 tons 14 KG RS21/ton RS20/ton Sand 250 tons 26 KG RS11/ton RS10/ton Material Usage Variance will be calculated as follows: Step 1: Calculate Standard Quantity Limestone: 10,000 units x 11 / 1000 = 110 tons Clay: 10,000 units x 14 / 1000 = 140 tons Sand: 10,000 units x 26 / 1000 = 260 tons
  • 25. 25 Step 2: Calculate the Variance Material Usage Variance = [Actual Quantity - Standard Cost (Step 2)] x Standard Price Limestone: (100 - 110) x RS70 = (RS700) Favorable Clay: (150 - 140) x RS20 = RS200 Adverse Sand: (250 - 260) x RS10 = (RS100) Favorable Total Usage Variance (RS600) Favorable Note: Actual price paid for the acquisition of materials shall be ignored since the variation between standard price is already accounted for in the material price variance. Analysis A favorable material usage variance suggests efficient utilization of materials. Reasons for a favorable material usage variance may include: Purchase of materials of higher quality than the standard (this will be reflected in adverse material price variance) Greater use of skilled labor Training and development of workforce to improve productivity Use and improvement of automated manufacturing tools and processes An adverse material usage variance indicates higher consumption of material during the period as compared with the standard usage. Reasons for adverse material usage variance include: Purchase of materials of lower quality than the standard Use of unskilled labor Increase in material wastage due to depreciation of plant and equipment
  • 26. 26 Direct Material Mix Variance Definition Direct Material Mix Variance is the measure of difference between the cost of standard proportion of materials and the actual proportion of materials consumed in the production process during a period. Direct Material Mix Variance: = Actual Quantity x Standard Price - Standard Mix Quantity x Standard Price = Standard Cost of Actual Mix - Standard Cost of Standard Mix = (Actual Mix Quantity - Standard Mix Quantity) x Standard Price As material mix variance is an extension of the material usage variance, the variance is based on the standard price rather than actual price since the difference between actual and standard material price is accounted for separately in the material price variance. Example Cement PLC manufactured 10,000 bags of cement during the month of January. Consumption of raw materials during the period was as follows: Material Quantity UsedStandard Mix Per BagActual Price Standard Price Limestone 100 tons 11 KG RS75/ton RS70/ton Clay 150 tons 14 KG RS21/ton RS20/ton Sand 250 tons 26 KG RS11/ton RS10/ton
  • 27. 27 Material Mix Variance will be calculated as follows: Step 1: Calculate the total consumption of raw materials Total Raw Materials Consumption (100 + 150 + 250) = 500 tons Step 2: Calculate the Standard Mix We need to calculate the quantity of each raw material which would have been consumed had the total usage of raw materials (500 tons) been based on the standard mix. Limestone: 500 tons units x11 / 51*=108 tons Clay: 500 tons units x14 / 51*=137 tons Sand: 500 tons units x26 / 51*=255 tons * Total Quantity under Standard Usage (11 + 14 + 26) = 51 KG per bag Note that the sum of the standard mix of raw materials calculated above equals the actual total consumption of 500 tons. This is because in material mix variance, we are not concerned about the efficiency of raw material consumption but rather their relevant proportions. Step 3: Calculate the Variance Material Usage Variance = [Actual Mix - Standard Mix (Step 2)] x Standard Price Limestone: (100 - 108) x RS70 = (RS560) Favorable Clay: (150 - 137) x RS20 = RS260 Adverse Sand: (250 - 255) x RS10 = (RS50) Favorable Total Usage Variance (RS350) Favorable
  • 28. 28 Explanation Material Mix Variance quantifies the effect of a variation in the proportion of raw materials used in a production process over a period. Material mix variance is a sub-division of material usage variance. While material usage variance illustrates the overall efficiency of raw material consumption during a period (in terms of the difference between the amount of materials which should have been used and the actual usage), material mix variance focuses on the aspect of proportion of raw materials used in the production process. Material mix variance is only suitable for performance measurement and control where the proportion of inputs to the production process can be altered without reducing the effectiveness of the final product. It may not therefore be used in industries that require a high degree of precision in the input variables such as in the pharmaceuticals sector. Analysis A favorable material mix variance suggests the use of a cheaper mix of raw materials than the standard. Conversely, an adverse material mix variance suggests that a more costly combination of materials have been used than the standard mix. A change in the material mix must also be analyzed in the context of other organization wide implications that may follow. Some of the effects a change in direct material mix include:  Change in the quality, performance and durability of the final product  Price offered by customers may vary as a result of a change in perceived quality of the product  Change in material mix may affect the workability of materials which may in turn affect labor efficiency
  • 29. 29 Direct Material Yield Variance Definition Direct Material Yield Variance is a measure of cost differential between output that should have been produced for the given level of input and the level of output actually achieved during a period. Direct Material Yield Variance: = (Actual Yield - Standard Yield) x Standard Material Cost Per Unit Example Cement PLC manufactured 10,000 bags of cement during the month of January. Consumption of raw materials during the period was as follows: Material Quantity UsedStandard Mix Per BagActual Price Standard Price Limestone 100 tons 11 KG RS75/KG RS70/KG Clay 150 tons 14 KG RS21/KG RS20/KG Sand 250 tons 26 KG RS11/KG RS10/KG Material Yield Variance shall be calculated as follows: Step 1: Calculate the Standard Yield for the total materials input 500 tons of materials should have yielded 9,804 bags Standard Yield = 500 tons x 1000 / 51 KG = 9,804 bags
  • 30. 30 Step 2: Calculate the Standard Cost of materials per bag Total material cost of 1 bag of cement: Limestone: 11 KGxRS70 =RS770 Clay: 14 KGxRS20 =RS280 Sand: 26 KGxRS10 =RS260 Total RS1,310 per bag Actual material price should be ignored since the variance between actual and standard price is accounted for in the material price variance. Step 3: Calculate the Total Yield Variance Material Usage Variance = [Actual Yield - Standard Yield (Step 1)] x Standard Cost / Unit (Step 2) Actual Yield - Standard Yield = 10,000 - 9,804 (Step 1) = 196 bags Total Material Yield Variance = 196 bags x RS1,310 (Step 2) = RS256,760 Favorable As the actual output achieved during the period is higher than the standard yield, the variance is favorable. Favorable material yield variance indicates the amount of savings in material costs as a result of better output yield than the standard.
  • 31. 31 Step 4: Calculate the Material Wise Yield Variances Individual material yield variance can be calculated in a similar way to the total yield variance. Materials: Actual Yield - Standard Yield (Step 3) x Standard Cost per bag (Step 2) =Yield Variance Limestone: 196 bags xRS770 =RS150,920 Clay: 196 bags xRS280 =RS54,880 Sand: 196 bags xRS260 =RS50,960 RS256,760 Note that sum of individual material yield variances equals the total yield variance calculated in step 3. Explanation Material Yield Variance measures the effect on material cost of a change in the production yield from the standard. Material yield variance is used in conjunction with material mix variance in order to provide additional analysis of the material usage variance. The difference between material usage and material yield variance is that the former focuses on the utilization of input at the start of production process whereas latter focuses on the efficiency in terms of the output yield during a period. Analysis A favorable material yield variance indicates better productivity than the standard yield resulting in lower material cost. Conversely, an adverse material yield variance suggests lower production achieved during a period for the given level of input resulting in higher material cost.
  • 32. 32 Conclusion Standard costing is a technique used for the purpose of determining standard cost and their comparison with the actual cost to find out the cause of difference between the two so that remedial action may be taken immediately. Direct material cost variance is the difference between the standard cost of materials specified for the output achieved, and the actual cost of direct materials consumed. Direct material price variance is the difference between actual price and standard price of materials consumed. Material usage variance is that portion of material cost which arises due to the difference between the standard quantity specified and the actual quantity used. Material mix variance may be defined as that portion of the material usage variance which is due to the difference between the standard and actual composition of material mixture. Material yield variance is calculated on the basis of output while the other variance are calculated on the basis of input. The variance is calculated as the difference between the standard output and actual output.